Question

Steven earned net income of $82080 and has fixed costs of $41040. Steven sold 12540 units for $30 each and has a 40% contribu
0 0
Add a comment Improve this question Transcribed image text
Answer #1
Sales (12540 x $30) $ 376,200.00
Less: Variable cost ( 1- 40% ) x 376200 $ 225,720.00
Contribution margin (40% x 376200 ) $ 150,480.00
Less: Fixed Cost $   41,040.00
Net operating Income $ 109,440.00
Less: Taxes (27360/109440) = 25% $   27,360.00
Net Income $   82,080.00
Option D is Correct (109,440 ; 25%)
Add a comment
Know the answer?
Add Answer to:
Steven earned net income of $82080 and has fixed costs of $41040. Steven sold 12540 units...
Your Answer:

Post as a guest

Your Name:

What's your source?

Earn Coins

Coins can be redeemed for fabulous gifts.

Not the answer you're looking for? Ask your own homework help question. Our experts will answer your question WITHIN MINUTES for Free.
Similar Homework Help Questions
  • please help me answer this question by 8:15pm today. thank you Edward earned net income of...

    please help me answer this question by 8:15pm today. thank you Edward earned net income of $83520 and has fixed costs of $41760. Edward sold 12760 units for $30 each and has a 40% contribution margin ratio. What were Edward's operating income and tax rate? O $139200: 40% O $313200; 27% O $194880; 38% O $111360: 25%

  • Net Income Planning Planning Holland Corporation earned an after-tax net income of $240,000 last year. Fixed...

    Net Income Planning Planning Holland Corporation earned an after-tax net income of $240,000 last year. Fixed costs were $1,200,000. The selling price per unit of its product was $120, of which $50 was a contribution to fixed cost and net income. The income tax rate was 40%. a. How many units of product were sold last year? 0 units b. What was the break-even point in units last year? 0 units C. The company wishes to increase its after-tax net...

  • 1) Key Company has a targeted sales volume of $62,300 units. Total fixed costs are $31,200....

    1) Key Company has a targeted sales volume of $62,300 units. Total fixed costs are $31,200. The contribution margin per unit is $1.20. What is targeted net income? A.) $31,200 B.) $43,560 C.) $37,440 D.) $74,760 2) ________ is the relative proportions or combinations of quantities of different products that comprise total sales. A) Sales mix B) Constant mix C) Fluctuating mix D) Variable cost ratio 3) The Todd Dolhun Company has the following information available: Targeted after-tax net income...

  • Sales (100,000 units) Variable costs Contribution margin Fixed manufacturing costs Operating income Interest Earnings before taxes...

    Sales (100,000 units) Variable costs Contribution margin Fixed manufacturing costs Operating income Interest Earnings before taxes Taxes (30°) Net Income $1.000.000 300.000 700,000 200,000 500.000 75.000 425.000 127,500 $297,500 Refer to the table. The degree of operating leverage S Select one: O a. 1.56x Ob.3.33% O c. 2.22% O d. 1.40x Sales (1,000 units) Variable costs Contribution margin Fixed manufacturing costs Operating income Interest Earnings before taxes Taxes (30°) Net Income Shares Outstanding $200,000 110,000 90,000 40.000 50.000 10,000 40,000...

  • Example 48: Fill in the effects of each of the following on breakeven point, operating income...

    Example 48: Fill in the effects of each of the following on breakeven point, operating income and net income using: 1 = increase, D = Decrease, NC = No Change. Breakeven point in units Net Income Breakeven point in dollars Operating Income Contribution Margin in dollars Contribution Margin Ratio Increase variable costs per unit Decrease variable costs per unit Increase variable costs in total Decrease variable costs in total Increase fixed costs Decrease fixed costs Increase selling price Decrease selling...

  • Carroll Corporation has two products, Q and P. During June, the company's net operating income was...

    Carroll Corporation has two products, Q and P. During June, the company's net operating income was $27,000, and the common fixed expenses were $58,000. The contribution margin ratio for Product Q was 40%. Its sales were $143.000, and its segment margin was $50,000. If the contribution margin for Product P was $48,000, the segment margin for Product P was: Multiple Choice $35,000 O $50,000 O O "ounle Corporation has two divisions: the South Division and the West Division. The corporation's...

  • Margin of safety ratio. 4. Net operating income ercentage. 10-13 Effect of taxes on break-even and...

    Margin of safety ratio. 4. Net operating income ercentage. 10-13 Effect of taxes on break-even and target volume Yosemite Enterprises desires to earn an after-tax income of $150,000 It has fixed costs of $1,000,000. a unit sales price o! $500, and unit variable cosEs of $200. The company is in the 30% tax bracket. 1. How many dol ars of sales revenue must be earned to achieve the after-tax profit of $150.000? 2. How many dollars of revenue would have...

  • Vaughn, Inc. sold 17,000 units last year for $190 each. Variable costs per unit were $48...

    Vaughn, Inc. sold 17,000 units last year for $190 each. Variable costs per unit were $48 for direct materials, $38 for direct labor, and $10 for variable overhead. Fixed costs were $35,000 in manufacturing overhead and $37.000 in nonmanufacturing costs. a. What is the total contribution margin? Total Contribution Margin b. What is the unit contribution margin? Unit Contribution Margin c. What is the contribution margin ratio? Contribution Margin Ratio % ho d. If sales increase by 6,000 units, by...

  • Last month, Laredo Company sold 620 units for $110 each. During the month, fixed costs were...

    Last month, Laredo Company sold 620 units for $110 each. During the month, fixed costs were $7,656 and variable costs were $66 per unit. Required: 1. Determine the unit contribution margin and contribution margin ratio. 2. Calculate the break-even point in units and sales dollars. 3. Compute Laredo's margin of safety in units and as a percentage of sales. Complete this question by entering your answers in the tabs below. Required 1 Required 2 Required 3 Determine the unit contribution...

  • APPLY THE CONCEPTS: Target income (number of units sold) Suppose a business has pricing and cost...

    APPLY THE CONCEPTS: Target income (number of units sold) Suppose a business has pricing and cost information as follows:: Price and Cost Information Amount Selling Price per Unit $10.00 Variable Cost per Unit $5.00 Total Fixed Cost $100 For the upcoming period, the company wishes to generate operating income of $400. Given the cost and pricing structure for the company’s product, how many units must the company sell to attain its target income? Remember that the basic equation for calculating...

ADVERTISEMENT
Free Homework Help App
Download From Google Play
Scan Your Homework
to Get Instant Free Answers
Need Online Homework Help?
Ask a Question
Get Answers For Free
Most questions answered within 3 hours.
ADVERTISEMENT
ADVERTISEMENT
ADVERTISEMENT